CAPM and expected Market Return

the equation below: Kce = Rf + B[E(Rm) - Rf] Rf= risk free rate B= Beta E(Rm)= expected market return In the Schweser notes the text states that E(Rm) is the expected market return but in the question on p.g. 54 Corpoate Finance #8 it asks for the companys CAPM but takes the market risk premium of 5% and uses this value for the E(Rm). The expected maket return was not given in this example. Any idea why schweser would you a risk premium for E(Rm)? Thanks

read again risk premium=[E(rm) - rf] and not E(rm).

thanks for the clarification!

They will try and trick you on the exam by giving you the Market Risk premium instead of the Market Return. Question will be as follows: XYZ Company is currently trading at xx a share. The RFR is 4%, the companies beta is 1.4 and the market risk premium is 10%. The stock is; The formula you want is E® = 4 + 1.4 [10] , but they want to catch those sleeping who will do E® = 4 + 1.4 [10 - 4]. Of course if you compute the formula with 10 - 4 that wrong computation will be an answer. Always be careful with CAPM questions, they are very easy but they try this trick alot.